Evaluate Double integral over triangular region

Click For Summary
The discussion focuses on evaluating a double integral over a triangular region defined by the vertices (0,0), (2,4), and (6,0). The integral in question is of the form ∫∫_D ye^x dA, with limits of integration identified as 0.5y to 6-y for x and 0 to 4 for y. A participant expresses difficulty in integrating the resulting expression, which simplifies to ∫_0^4 (ye^(6-y) - ye^(y/2)) dy. Another contributor suggests using integration by parts and breaking the integral into two separate parts to facilitate the solution.
nlsherrill
Messages
320
Reaction score
1

Homework Statement


Evaluate the double integral.

[I don't know how to write it in latex, sorry, but its the double integral over the region 'd' of ye^x dA

D is the triangular region with vertices (0,0), (2,4), and (6,0).


Homework Equations





The Attempt at a Solution



So the limits of integration I have are .5y to 6-y with respect to x, and 0 to 4 with respect to y. I think this is right. The farthest I can get is:

Integral 0..4 ye^6-y - ye^.5y dy

I can't figure out how to integrate these terms. Any hints?
 
Physics news on Phys.org
nlsherrill said:

Homework Statement


Evaluate the double integral.

[I don't know how to write it in latex, sorry, but its the double integral over the region 'd' of ye^x dA

D is the triangular region with vertices (0,0), (2,4), and (6,0).


Homework Equations





The Attempt at a Solution



So the limits of integration I have are .5y to 6-y with respect to x, and 0 to 4 with respect to y. I think this is right. The farthest I can get is:

Integral 0..4 ye^6-y - ye^.5y dy

I can't figure out how to integrate these terms. Any hints?
Here's your integral in LaTeX:
\int_{y = 0}^4 (ye^{6 - y} - ye^{y/2})dy

Break this up into two integrals. Note that e6 - y = e6e-y

Integration by parts should work on both integrals.
 
Thanks! forgot about integration by parts.
 
Question: A clock's minute hand has length 4 and its hour hand has length 3. What is the distance between the tips at the moment when it is increasing most rapidly?(Putnam Exam Question) Answer: Making assumption that both the hands moves at constant angular velocities, the answer is ## \sqrt{7} .## But don't you think this assumption is somewhat doubtful and wrong?

Similar threads

  • · Replies 19 ·
Replies
19
Views
2K
Replies
3
Views
2K
Replies
10
Views
2K
Replies
2
Views
2K
  • · Replies 10 ·
Replies
10
Views
2K
Replies
4
Views
2K
  • · Replies 3 ·
Replies
3
Views
2K
  • · Replies 17 ·
Replies
17
Views
4K
  • · Replies 7 ·
Replies
7
Views
2K
  • · Replies 5 ·
Replies
5
Views
2K